LSAT and Law School Admissions Forum

Get expert LSAT preparation and law school admissions advice from PowerScore Test Preparation.

 biskam
  • Posts: 124
  • Joined: Aug 18, 2017
|
#39461
Steve Stein wrote:Hi Kristina,

Thanks for your response. Again, if we take away the word "only" from Luis' statement, then he would support programs that are "expected to yield practical applications."

The proposed study has the stated purpose of developing new seed breeds--does that mean that the study should be expected to yield practical applications? I'm not entirely sure what Luis would have to say about that, but if we presume that this study falls under Luis' category of programs that are "expected to yield practical applications," then Luis would (again, if we were to remove the word "only" away from his statement) disagree with answer choice (B).

I hope that's helpful! Please let me know whether this is clear--thanks!

~Steve
I've read this entire thread and I want to make sure I understand correctly WHY we don't know for sure whether Luis would disagree with B... is it because we can't know for sure if X has the "purpose" of doing something that we can "expect" it to do something? That this uncertainty is what makes us say we don't know if he will disagree?
 Eric Ockert
PowerScore Staff
  • PowerScore Staff
  • Posts: 164
  • Joined: Sep 28, 2011
|
#39767
Hi!

Thanks for the follow up. I think you can get into the weeds a bit with answer choice (B) trying to figure out whether Luis would truly disagree or not. But frankly, when it comes to these Point at Issue answers (and really this works for the answers to most question types), I like to look for the most obvious problem with an answer. Here, if you can show that you don't know what either one of the authors' feels about an answer, that's enough to show it's wrong.

Here, Cynthia undoubtedly believes the government should fund projects that do try to expand our theoretical knowledge. But that doesn't mean she necessarily agrees that the government should not fund projects that don't. So, with (B), you can't really find Cynthia agrees with that statement. That could really be the end of the analysis because without one author agreeing with the answer choice, it can't be a Point at Issue.

If you want to get into the analysis for Luis (which you can avoid here, as I mentioned above), things are a bit more complex. His belief is that if research is to be funded, then it MUST be expected to yield practical benefits. Alternatively (via the contrapositive), if the research is NOT expected to yield practical benefits, it should not be funded.

This answer seems like there is an expectation of practical applications (after all, that is the purpose of the research), but it is a bit unclear. But, no matter what, one of two things happens here:

1. There is an expectation of practical benefit.
Luis only says this is a requirement for funding not a guarantee. So, in that case, we don't know what Luis thinks about this answer. So it is wrong.

2. There is not an expectation of practical benefit.
Luis would then AGREE with this answer that the research should not be funded. Then, we would have to know that Cynthia DISAGREES with this answer. But we don't know that. So this answer is wrong.

Either route you take here, or the simple route above, leads you to the same place. Answer choice (B) can't be proven as a point of disagreement.
User avatar
 lsatquestions
  • Posts: 66
  • Joined: Nov 08, 2021
|
#96751
Hi, I selected the correct answer with B as a contender. With your explanation if makes more sense why B is incorrect, but upon reading the forum I now have A as a contender. If there is an expectation of practical benefit, Luis would disagree with A because the practical applications of the research are "unforseen." Cynthia categorically believes that the government should fund theoretical research. Is A incorrect because the reason she believes this is not what is state in the answer choice?
 Robert Carroll
PowerScore Staff
  • PowerScore Staff
  • Posts: 1787
  • Joined: Dec 06, 2013
|
#97419
lsatquestions,

Answer choice (A) says "because". As you said, Cynthia believes theoretical research should be funded because of the knowledge one would gain. She does NOT think it should be funded because of the practical applications. So she would agree that pure theoretical research should be funded, but not because of the applications. Answer choice (A) says "because" of the applications, so Cynthia doesn't agree with that, nor does Luis.

Robert Carroll

Get the most out of your LSAT Prep Plus subscription.

Analyze and track your performance with our Testing and Analytics Package.